Quiznetik

Important Questions Repeated in MD Exams | Set 3

1. Which of the following proto-oncogenes is least likely to be associated with an increased risk of breast cancer?

Correct : E. Wiskott Aldrich Syndrome

2. A 40-year-old male is operated on for a medullary carcinoma of the thyroid. He expresses concern about his 10 years son getting the disease. The best test:

Correct : D. DNA analysis for RET proto-oncogene in peripheral blood

3. Patient with germ cell tumor receiving BEP. Which side-effect would necessitate cessation of the responsible drug:

Correct : A. neutropenia

4. A female presents with shortness of breath. CXR is shown with whiteout of Left lung. Told neurone-specific enolase +ve, next best step in management:

Correct : C. Chemotherapy

5. Which of the following best describes the incidence of colon cancer with age:

Correct : D. Exponential rise after age 50

6. All of the following can occur in germ line leading to breast cancer except which one:

Correct : C. Bcl-2

7. Which of the following is the least likely to be involved in hereditary colon cancer:

Correct : E. Chromosomal translocation

8. A 50 year patient presents with metastatic renal cell carcinoma. The best management would be:

Correct : B. IL-2

9. The genetic defect in hereditary non-polyposis coli lies in:

Correct : C. DNA repair defect

10. A female with a history of breast cancer removed. Examination revealed previous modified radical mastectomy scar. Diffuse erythema, with nodules and plaquing over and around scar with some extension onto contralateral breast is evident. The most likely diagnosis is:

Correct : C. Tumor recurrence

11. Which is the strongest predictor of an increased risk of breast cancer in a 40 ys woman:

Correct : C. Previous breast carcinoma in contralateral breast

12. A young male has non-Hodgkin's lymphoma. The LMO starts prednisone whilst awaiting a haematology admission. He presents unwell with WCC previously 30, now 2, urate 2.3, K 8.0, phosphate 10, Cr .25. The most likely diagnosis?

Correct : B. Tumor lysis syndrome

13. A young female (30 years) had mantle radiotherapy in her teens. The most likely cancer site in her now:

Correct : B. Breast

14. The most common mechanism of spread of epithelial ovarian cancer is:

Correct : C. Adjacent structures

15. A man with metastatic prostate cancer with increasing bone pain. The decision is made to start a GnRH agonist Your advice regarding the likely effect of this:

Correct : B. Initial worsening of symptoms over two weeks followed by gradual resolution

16. Concerning lung cancer:

Correct : B. The use of chemotherapy in combination with radiotherapy in patients with stage III non-small cell lung cancer is associated with improved survival.

17. Regarding familial breast cancer, which of the following statements is true?

Correct : B. mutation of the BRCA1 gene may be associated with an increased risk of both breast and ovarian cancer.

18. BRCA-1:

Correct : C. Associated with breast and ovarian cancer

19. Oesophageal cancer is associated with:

Correct : E. Smoking

20. Small cell lung cancer is more associated with ( than NSCLC)

Correct : E. Hyponatraemia

21. Regarding cell kinetics of cancer:

Correct : A. 1 cm tumor = 109 cells

22. 73 years female with metastatic breast cancer, on tamoxifen for 6/12. 2/7 Hx of nausea and thirst. Na+ 155, K+ 5.2, urine osm 137. Has multiple mets including sella turcica. Next most appropriate treatment:

Correct : B. Fluid restriction

23. Pre-menopausal woman found to have left axillary and supraclavicular nodes. Biopsy showed poorly differentiated cancer, oestrogen receptor poor. She has no other Symptoms, examination is otherwise normal and CXR and mammogram are normal. Next step:

Correct : B. Bronchoscopy

24. A patient with known disseminated lymphoma is anaemic, with Hb = 100, MCV = 74, Fe = 4, Ferritin = 300. This is best explained by:

Correct : C. Reduced erythropoietin

25. Radiotherapy has its most severe effects on which tissues?

Correct : D. bone marrow

26. Which of the following is more likely to result from the administration of bleomycin?

Correct : A. sodium urate

27. Patient with metastatic bowel cancer is taking Codeine 30 mg q4h without adequate analgesia. What is the most appropriate therapy?

Correct : A. continue Codeine and add Paracetamol 1g qid

28. Concerning the prophylactic use of G-CSF during cancer chemotherapy:

Correct : B. it decreases the severity of the neutropenic nadir

29. Phase 2 trial of new cancer drug. Which is least likely to account for variation in results:

Correct : C. Gender

30. Breast cancer, least likely defect:

Correct : C. Bcl-2

31. In women the biggest risk factor for breast cancer is:

Correct : B. Previous contra lateral breast cancer

32. Best treatment of metastatic renal cell cancer:

Correct : A. Sutinib

33. Major spread of ovarian cancer

Correct : A. Adnexal

34. Patient gets bleomycin, etoposide, and cisplatin. Which cause for cessation of specific drug?

Correct : B. Neutropenic sepsis

35. 55 year female - breast cancer 3 yrs ago had mastectomy only Presents with lower back pains. Some neurology - mild --> equivocal plantar. MRI Shows lesions in 3- 4 different vertebra - partially compressing spinal cord.

Correct : A. surgical decompression and radiotherapy

36. Cisplatin based chemotherapy. Which anti-emetic is most effective >24 post chemotherapy:

Correct : B. Ondansetron

37. Least effective for identifying an unknown primary

Correct : C. Mediastinal undifferentiated carcinoma

38. Middle aged smoker. Whole of left lung – white, collapse. Bronchoscopy shows an undifferentiated carcinoma in the left main bronchus. Presents with cough and haemoptysis. Small left clavicular lymph node palpable. Treatment:

Correct : C. Chemotherapy

39. Smoker with carcinoma of the lung. Well until recently. Bronchoscopy shows extrinsic compression of the bronchus. Low serum sodium. Diagnosis:

Correct : A. Small cell carcinoma

40. In a young man testicular malignancy, how does  HCG cause gynaecomastia?

Correct : C. By increased oestrogen

41. Young lady had radiotherapy for Hodgkin‟s disease. 10 years later, most likely sequelae:

Correct : D. Carcinoma breast

42. DNA microsatellite - best description

Correct : A. Single locus 2 - 4 base pairs repeats

43. Which drug is least likely to affect cytokine production - especially IL-2 in T cells?

Correct : B. Mycophenalate

44. Among viruses which are activated due to immunosuppression the least oncogenic virus is

Correct : B. CMV

45. Disease least likely to have a family predisposition

Correct : B. Lung cancer

46. Which one of the following drugs has the least bone marrow suppression?

Correct : E. Vincristine

47. Most common mode of spread of epithelial ovarian tumors occurs to

Correct : B. Adnexae

48. Changes least likely with familial colonic cancer

Correct : C. Chromosomal translocation

49. Small cell lung carcinoma can cause (?all of the following except)

Correct : A. Hypercalcemia

50. BRCA-1

Correct : D. Associated with breast & ovarian carcinoma

51. Predisposing factors to oesophageal adenocarcinoma include the following except

Correct : A. Achalasia

52. Young woman with amenorrhoea & large uterus. CXR shows rounded opacities. What is the best test to monitor this condition?

Correct : C. -fetoprotein

53. How would you treat metastatic Ca Breast in a postmenopausal woman?

Correct : A. Anti-oestrogen

54. What is the best prognostic indicator of post op survival for Ca Breast?

Correct : D. Tumour grade/ stage

55. Which malignancy is least amenable to treatment?

Correct : B. Adeno Ca with sclerotic lesion in bone

56. Associations for 1o Ca can be made for all of the following except

Correct : D. Mediastinal Adeno Ca in a young man

57. Mechanism of drug resistance of chemotherapeutic drugs

Correct : A. P glycoprotein

58. A middle-aged man has a melanoma fully excised. What is the most important reason to follow him closely?

Correct : A. Local and systemic recurrence

59. A 68 year old man is admitted to the ER having had three large maroon colored stools. On arrival, he passes more bloody stools and clots. He is pale, orthostatic and tachycardic. NG aspirates are bilious. After resuscitation is begun, which of the following is the most appropriate initial test?

Correct : C. Rigid proctoscopy

60. The most common mode of spread of colon cancer is

Correct : B. Lymphatic

61. The following is the most important prognostic determinant of survival after treatment for colorectal cancer?

Correct : A. Lymph node involvement

62. A 68-year-old man presents to his primary care physician for a routine physical examination. The patient‟s medical history is significant for hypertension. The patient is found to have guaiac-positive stools and is subsequently referred for colonoscopy. Colonoscopy reveals a “golf ball”-size, near-obstructing tumor in the descending colon, not admitting the scope. The biopsy is positive for adenocarcinoma of the colon. Which of the following is the next step in the management of this patient?

Correct : D. Do metastatic work up, but plan for colon resection anyway

63. For previous patients, after the appropriate evaluation, the patient undergoes surgery. No intraoperative evidence of metastases is identified. Postoperatively, the pathology report reveals that the tumor is an adenocarcinoma invading into the pericolonic fat, with 2 involved lymph nodes. After the patient recovers from surgery, which of the following is the most appropriate next step in his management?

Correct : C. Chemotherapy with 5-fluorouracil (5-FU) based regimen

64. A 65-year-old woman with no significant past medical history presents to the emergency department with a 2-day history of left lower quadrant abdominal pain. The patient denies nausea and vomiting, although she claims decreased oral intake. She also reports a low-grade fever and mild diarrhea. She describes a milder episode several years ago, which resolved on its own. On physical examination, the patient is found to have left lower quadrant tenderness with some mild guarding, but no rebound. She is hemodynamically stable, and her heart rate is 82 per minute. In the initial management of this patient, which of the following is the most sensitive diagnostic test?

Correct : D. Abdominal/pelvic CT with oral contrast

65. A 60-year-old man presents for an annual physical examination. The examination is normal except for a palpable mass in the rectum on digital rectal examination. The patient denies any change in bowel habits and feels well. Rectal cancer is suspected. What is the next best step in the evaluation of this patient?

Correct : D. Full colonoscopy with biopsy of the lesion

66. A 78-year-old woman with coronary artery disease and severe chronic obstructive pulmonary disease is admitted to the hospital with painless jaundice. CT scan reveals the presence of multiple lesions in the liver, suggestive of metastases, and a nearly obstructing upper rectal mass. Colonoscopy demonstrates a large, ulcerated tumor in the proximal rectum and a residual lumen of less than 1 cm in diameter. While in the hospital, the patient develops a large bowel obstruction. What is the best treatment modality for this patient?

Correct : E. Placement of a rectal self-expanding metal stent

67. A 57-year-old man is found to have a rectal mass 3 cm from the anal verge on digital rectal examination. Subsequent colonoscopy and biopsy confirm rectal adenocarcinoma. EUS examination demonstrates penetration of the tumor into, but not through, the muscularis propria, but shows significant perirectal lymph nodes. CT scan of chest/abdomen/pelvis demonstrates no metastases. The patient is staged as T2N1M0. What procedure should be attempted to remove the primary lesion in this patient?

Correct : E. Neo-adjuvant chemoradiation followed by abdominoperineal resection (APR)

68. A 70-year-old man is found to have distal rectal cancer during a screening colonoscopy. The patient undergoes preoperative staging and is found to have a 1.5-cm rectal mass that does not invade the muscularis propria of the rectal wall. There is no regional lymphadenopathy and no evidence of distant metastases. The patient is staged at T1N0M0. The patient is advised to undergo APR but refuses because it will lead to anal sphincter loss and permanent colostomy. Which of the following represents a viable alternate therapeutic option for this patient?

Correct : D. Full-thickness surgical removal of tumor (transanal excision)

69. Which of the following may be appropriate initial therapy for a 4 cm cancer of the anal canal?

Correct : C. Combined chemotherapy and radiotherapy

70. Which of the following processes is most likely to be associated with apoptosis?

Correct : A. Nucleosomal DNA laddering

71. The purpose of a phase II clinical trial is to:

Correct : B. estimate the efficacy of treatment to be tested.

72. Which of the following chemotherapy agents is most effective against soft tissue sarcoma?

Correct : D. Doxorubicin

73. What number of axillary lymph nodes removed for sampling during a level I/ II node dissection for breast cancer most likely would require postoperative radiation therapy to the axilla?

Correct : A. 02

74. Which of the following types of cancer was associated with a statistically significant improvement in survival for patients who had one to three solitary brain metastases and received whole brain radiation therapy with the addition of stereotactic radiosurgery, according to results of the RTOG 9508 phase III trial?

Correct : C. Non-small cell lung cancer

75. In RTOG 8501, what was the 5-year overall survival rate for patients who had esophageal cancer and received radiation therapy alone?

Correct : A. 00%

76. Which of the following tumor stages is correctly matched to the appropriate prostate cancer involvement or invasion?

Correct : A. Stage T2b is associated with tumor involvement of more than one half of one lobe but not both lobes of the prostate.

77. According to GOG 122 (Randall), the survival rate of patients who have surgical stage III-IV endometrial cancer and receive adjuvant abdominopelvic irradiation compared to adjuvant chemotherapy is:

Correct : A. worse.

78. A well-lateralized, stage T3N0 squamous cell carcinoma of the nasopharynx has at least a 15% risk of metastasis to the:

Correct : C. posterior triangle lymph nodes bilaterally.

79. What is the second leading cause of lung cancer?

Correct : D. Exposure to radon

80. Which of the following chromosomal translocations is most commonly associated with Burkitt lymphoma?

Correct : A. t(8;14)

81. What is the most common secondary malignant neoplasm inside the radiation field for patients with retinoblastoma?

Correct : D. Osteosarcoma

82. Three different lung cancer radiation treatments are being assessed for maximum- tolerated dose. This study is an example of what type of clinical trial?

Correct : A. Phase I

83. Which of the following factors is the most important predictor of disease-free survival and overall survival in patients with breast cancer?

Correct : C. Axillary lymph node status

84. Which of the following diagnostic tests is most useful for determining the tumor stage of esophageal cancer?

Correct : B. Endoscopic ultrasound

85. Which of the following studies compared surgery alone versus surgery plus external-beam pelvic irradiation in patients with early-stage endometrial cancer and required all patients to undergo surgical staging with pelvic lymphadenectomy?

Correct : A. GOG 99 (Keys)

86. Which of the following findings is most likely to be caused by loss of function of right cranial nerve XII?

Correct : B. Deviation of the tongue to the right side with protrusion

87. Which of the following statements about video-assisted thoracic surgery for patients with lung cancer is FALSE?

Correct : D. Locoregional recurrence is increased.

88. According to GOG 99 (Keys), which of the following sites is most likely to be associated with cancer recurrence in a patient who has early-stage endometrial cancer and undergoes TAH-BSO without adjuvant irradiation?

Correct : B. Vaginal vault

89. Which of the following laboratory tests is most appropriate for determining if a patient with acromegaly has responded favorably to radiation therapy?

Correct : D. Insulin-like growth factor-1 (IGF-1)

90. Which of the following soft tissue sarcomas is most likely to be found in a patient with neurofibromatosis, type 1?

Correct : D. Malignant peripheral nerve sheath tumor

91. Which of the following survival times is most likely to be associated with a patient who has a class II brain metastasis, based on the Radiation Therapy Oncology Group (RTOG) recursive partitioning analysis (RPA)?

Correct : C. 04.2 months

92. Radiation-induced cell death in carcinomas typically occurs immediately after attempting to traverse which phase of the cell cycle?

Correct : D. M

93. Which of the following antigens is a T-cell marker?

Correct : A. CD7

94. Which of the following statements about adenocarcinoma of the lung is true?

Correct : A. It is the most common histologic subtype.

95. A 9-month-old infant has a mass in the left flank, periorbital ecchymosis, and bone metastasis to the right femur. Biopsy results are consistent with neuroblastoma. What cancer stage would be most appropriate for this patient's condition?

Correct : C. Stage 4

96. Hypoxia adversely affects the treatment and management of cancers because it:

Correct : C. activates molecular pathways contributing to resistance.

97. What is the best method for minimizing bias in a national trial comparing two arms therapy regimens?

Correct : A. Randomization

98. Which of the following primary bone tumors is most common?

Correct : D. Osteosarcoma

99. Which of the following complications is NOT associated with a radical prostatectomy?

Correct : A. Proctitis

100. Which of the following FIGO stages is associated with a patient who has endometrial cancer with extension to the cervical stroma?

Correct : C. IIB